LSAT and Law School Admissions Forum

Get expert LSAT preparation and law school admissions advice from PowerScore Test Preparation.

User avatar
 Dave Killoran
PowerScore Staff
  • PowerScore Staff
  • Posts: 5852
  • Joined: Mar 25, 2011
|
#43483
Complete Question Explanation
(The complete setup for this game can be found here: lsat/viewtopic.php?t=7431)

The correct answer choice is (A)

If G is not assigned to a team, then each team features one experienced plumber and one inexperienced plumber. From the fourth rule, T must be assigned to K. F is the next logical plumber to examine due to the number of restrictions on F, and of the remaining inexperienced plumbers F must be assigned to a team with S. Because J cannot be assigned to a team with R, J must be assigned to a team with V, leaving M to team with R:
J95_Game_#4_#23_diagram 1.png
Answer choice (A) is thus correct.
You do not have the required permissions to view the files attached to this post.

Get the most out of your LSAT Prep Plus subscription.

Analyze and track your performance with our Testing and Analytics Package.